This contest has ended on 3rd september
TOPICS
Relation and function, matrix and determinants, vector, system of equations, sequence and series, permutation and combination, binomial theorem, complex numbers, polynomials, inequalities .
Rules
I will post the first problem. If someone solves it, he or she can post a solution and then must post a new problem.
A solution must be posted below the thread of the problem. Then, the solver must post a new problem as a separate thread.
Please make a substantial comment.
Make sure you know how to solve your own problem before posting it, in case no one else is able to solve it within 24 hours. Then, you must post the solution and you have the right to post a new problem.
If the one who solves the last problem does not post a new problem in 2 hours, the creator of the previous problem has the right to post another problem.
It is NOT compulsory to post original problems. But make sure it has not been posted on Brilliant.
If a diagram is involved in your problem please make sure it is drawn by a computer program.
Format your solution in , picture solution will be accepted but only picture will not be, i.e. you can use picture for diagram or something, but not for the complete solution. Also make sure your solution is detailed and make sure to proof all claims.
Do not post problems on same topic frequently like 3 continuous inequality problem are not allowed. 2 are sufficient.
For those who are on slack Please post in #general that new problem is up along with problem number and the link to contest (https://brilliant.org/discussions/thread/algebra-contest/?sort=new)
Handwritten picture solution are accepted only when they are easily understandable!!
Easy Math Editor
This discussion board is a place to discuss our Daily Challenges and the math and science related to those challenges. Explanations are more than just a solution — they should explain the steps and thinking strategies that you used to obtain the solution. Comments should further the discussion of math and science.
When posting on Brilliant:
*italics*
or_italics_
**bold**
or__bold__
paragraph 1
paragraph 2
[example link](https://brilliant.org)
> This is a quote
\(
...\)
or\[
...\]
to ensure proper formatting.2 \times 3
2^{34}
a_{i-1}
\frac{2}{3}
\sqrt{2}
\sum_{i=1}^3
\sin \theta
\boxed{123}
Comments
PROBLEM 1
Find the number of ordered pairs of real numbers (a,b) such that
(a+bi)2002=a−bi
Here i=−1
Log in to reply
SOLUTION TO PROBLEM 1 let a+bi=eix then e2002ix=e−ix and e2003ix=1 so all te possible a+bi are 2003th roots of unity . we know from here that a+bi has 2003 diffent solutions and for each solution we get a different ordered pair for (a,b) and hence the number of ordered pairs (a,b) is 2003
Log in to reply
Right. Post next
Log in to reply
Log in to reply
Problem 22
Passed on from Jesse Nieminen, since he didn't wish to post.
Let a,b,c∈R be real numbers such that a+b+c>0. Prove the inequality
a3+b3+c3≤(a2+b2+c2)23+3abc
Log in to reply
(a3+b3+c3−3abc)2=[(a+b+c)(a2+b2+c2−ab−bc−ca)]2=(a2+b2+c2+2ab+2bc+2ca)(a2+b2+c2−ab−bc−ca)2≤(a2+b2+c2)3⟹a3+b3+c3≤(a2+b2+c2)23+3abc.□
(a2+b2+c2+2ab+2bc+2ca)(a2+b2+c2−ab−bc−ca)2≤(a2+b2+c2)3
is true because if we substitute x=a2+b2+c2,y=ab+bc+ca, we get (x+2y)(x−y)2≤x3⇔2y≤3x⇔2ab+2bc+2ca≤3a2+3b2+3c2⇔a2+b2+c2+(a−b)2+(b−c)2+(c−a)2≥0
PROBLEM 9
Let a>0 be a real number and f(x) a real function defined on all R satisfying for all x belong to R
f(x+a)=21+f(x)−f(x)2
Prove that f(x) is periodic and give an example for f(x) when a=1.
Log in to reply
Replace x by x+a, 2 times you will find that f(x+3a)=f(x+a). Hence it is periodic with period 2a. Eg. f(x)=(2+√6)/4.
Log in to reply
Absolutely right. Noww post next
Problem 17
Find real numbers a, b and c such that
∣ax+by+cz∣+∣bx+cy+az∣+∣cx+ay+bz∣=∣x∣+∣y∣+∣z∣
for all real numbers x, y and z.
Log in to reply
Unordered triplets are (1,0,0) and (-1,0,0).
Log in to reply
Post solution too
Log in to reply
Log in to reply
problem 2 given the roots of y8−y7+y6−1=0 are y1,y2,...,y7,y8 find the value of y191+y291+...+y791+y891
Log in to reply
Solution to Problem 2
y8−y7+y6−1=0
Now we have to form a polynomial whose roots are y11,.....
Now, let x=y1
then y=x1
Putting it in the above equation, we get
x8+x2−x+1=0
x8=x−x2−1
x9=x2−x3−x...(1)
Putting x=y11,...,y81 repeatedly in the equation and add them all and using bit Newton Sum (S1=S2=S3=0)
And answer is 0.
Log in to reply
Post the next question.
you are right.
Is thE Answer 0?
Problem 3
Let a,b,x,f(x) be positive integers such that when a>b, f(a)>f(b).
If f(f(x))=x2+2, find the value of f(3)
Log in to reply
Log in to reply
Its right. Post next
Post next soon
Problem 4
Let a,b lie in first quadrant and let x be an acute angle.
Given that (acosx)2+(bsinx)2=[(sin2x)(2a+b)]2
Find the value of tanx.
Log in to reply
If we denote the three vertices by A(a,b),B(acscx,0),C(0,bsecx) then observe that ∣AB∣2=a2cot2x+b2,∣BC∣2=a2+b2tan2x,∣CA∣2=a2csc2x+b2sec2x
which in turn shows ∣AB∣2+∣BC∣2=∣CA∣2 and so B=2π.
Now, the circumcentre of a right angled triangle lies at the mid-point of it's hypotenuse and so the co-ordinates of the circumcenter is (M,N)=(2acscx,2bsecx)
From the relation we have , a2cos2x+b2sin2x=sin2xcos2x(a+b)2
⟹(acos2x)2+(bsin2x)2−2absin2xcos2x=0⟹acos2x=bsin2x
So we have , cscx=aa+b,secx=ba+b
So the circumcenter is : (2a2+ab,2b2+ab)
Log in to reply
Correct @Aditya Sharma but sorry I rephrased the problem so @Prince Loomba will decide who will post the next problem.
You post the next problem. The original was this only
Wait @Harsh Shrivastava , you can't change the problem for the reason someone can't do or have problems with that, the solution is done. Let the problem be back to it's original state
Log in to reply
You post the next prob.
Post the next
Circumvented? And is this in the topic list at the starting of note?
Log in to reply
Corrections done.
If I tell the name of the topic, the problem will be trivial :)
I got tanx=a/b. Idk how to find circumcentre
Log in to reply
Lemme rephrase the problem.
Done post the solution.
Log in to reply
Problem 5:
Find all functions f:Z→Z such that f(m+n)+f(m)f(n)=f(mn+1) where m,n are integers.
Log in to reply
Is the answer only 1? (f(x) = 0)
Log in to reply
NO, f(x)=0 is trivial, And more functions are there.
x-1 is also
Log in to reply
I got a weird thing. All of the functions of the type x2n−1 are satisfying according to me. Please check.
Log in to reply
hmm ri8 but there are some complicated functions involving mod also. anyways post the solution otherways
Log in to reply
f(x)=0 if x≡1,3(mod4), f(x)=1 if x≡2(mod4), f(x)=−1 if x≡0(mod4).
Mod function: (Alternatively the trig function)Log in to reply
Two functions f (x)=0 and f (x)=x-1
Log in to reply
There are 6, excluding the zero function
Log in to reply
x2−1
Log in to reply
Log in to reply
f(x)=-cos((2n+1)×pi×x/2)
f (x)=sin (n×pi×x)
Log in to reply
Firstly the floor is meaningless here since it's a function from integers to integers. And moreover you are not supposed to guess functions, you have to show the derivation. Otherwise it's not a solution
One more f (x)={x} fractional part. 6 are done
f (x)=[x]-1 (floor function)
There are infinite functions I can make
Log in to reply
yes it will work , i wll ppst other functions later . for now you may post the next question
Is there any mistakes I made?
Problem 26
Can someone please post a solution to my problems!!!
Prove for positive x, y and z
x2+y2−xy+y2+z2−yz≥z2+x2+zx
Log in to reply
Post easy, I will solve haha
Log in to reply
This is very, very simple. You just have to think outside the box, and you will get it.
Take a triangle ABC and construct a point P inside the triangle such that angleAPB = angleBPC = angleCPA= 120°.
Now by law of cosines, we can let AP = x,BP = y and CP = z.
The terms in the inequality represents side lengths of triangle ABC and thus the inequality is true by triangle inequality.
Problem 6
Let S1,S2,S3 denote the sum of n terms of 3 arithmetic progressions with first terms unity and their common differences are in harmonic progression. Prove that
n=S1−2S2+S32S3S1−S1S2−S2S3.
Log in to reply
I would like to cut the algebraic bash out of my solution, which I have, but it is quite a standard expansion.
S1S2S3d1d2d3a⟹S1S2S3=2n(2a+(n−1)d1)=2n(2a+(n−1)d2)=2n(2a+(n−1)d3)=x1=x+k1=x+2k1=1=n+2rn−1=n+2r+2kn−1=n+2r+4kn−1
Substituting back into the original expression, after much painful expansion, you get the fraction to simplify to n.
For those who want the expansion:
(n+2rn−1)−2(n+2r+2kn−1)+(n+2r+4kn−1)2(n+2rn−1)(n+4k+2rn−1)+(n+2rn−1)(n+2r+2kn−1)+(n+2r+2kn−1)(n+2r+4kn−1)=r(k+r)(2k+r)k2(n−1)2(n+2rn−1)(n+4k+2rn−1)+(n+2rn−1)(n+2r+2kn−1)+(n+2r+2kn−1)(n+2r+4kn−1)=k2(n−1)r(2(n+2rn−1)(n+4k+2rn−1)+(n+2rn−1)(n+2r+2kn−1)+(n+2r+2kn−1)(n+2r+4kn−1))(k+r)(k+2r)=k2(n−1)r(r(k+r)(2k+r)k2n(n−1))(k+r)(k+2r)=k2(n−1)k2n(n−1)=n
Log in to reply
This is not the solution. The solution is 10 times easier. I will consider it as solution either you solve full or take out the shortcut that I have
Log in to reply
@Prince Loomba
Can you please post the easy solution.Problem 7
Let Hn=11+21+…+n1. Prove that for n≥2
n(n+1)n1<n+Hn
Log in to reply
Note that n+Hn=(1+11)+(1+21)+…+(1+n1) (These brackets are all distinct terms). Now, by AM-GM, we have
ni=1∑n(1+i1)nHn+nHn+n>ni=1∏n(1+i1)>nn+1>n(n+1)n1
Thus, proven.
Problem 8
Let n be a positive integer. Prove (x2+x)2n+1 is irreducible.
Log in to reply
look at the polynomial mod 2
its equivalent to (x2+x+1)2n
so if (x2+x)2n+1 ≡ f(x) g(x) mod 2
f(x)=(x2+x+1)m mod 2
g(x)=(x2+x+1)(2n−m) mod 2
f(x)=(x2+x+1)m+2P(x)
g(x)=(x2+x+1)(2n−m)+2Q(x)
f(x)∗g(x)=((x2+x+1)m+2P(x))((x2+x+1)(2n−m)+2Q(x))
Let x=w (complex cube root of unity)
(w2+w)2n+1=2P(w)∗2Q(w)
2 = 4P(w)Q(w)
We got P (w)Q (w) as rational numbers but they were integer polynomials.
Hence a contradiction!
Let a∈I be a root of it. (a2+a)2n=−1. Clearly for n>=1, LHS is a perfect square with range >=0, so inequality cant hold
Thus no integer can be its root.
Hence proved
Log in to reply
Ya same way About to post it But u did it first
Log in to reply
Log in to reply
Not quite. Your method is incorrect since irreducible just means that it cannot be factored. A property of irreducible polynomials (degree > 1) is that they have no integral roots, but the converse isn't true.
It sufficies to consider the equivalent rational polynomial
(x2−1)2n+42n
It is known that every non-trivial binomial coefficient in the rows of Pascal's Triangle which are powers of 2 are all even.
Thus, in the expasion of the above polynomial, only the leading co-efficient and the constant are not even.
The leading coefficient is clearly not divisible by 2.
The constant term is clearly not divisible by 4.
Therefore, Eisenstein's Criterion holds and the polynomial is irreducible.
Hence, the original polynomial is also irreducible.
Problem 10
Find the number of polynomials of degree 5 with distinct coefficient from the set
{1,2,...9}
that are divisible by x2−x+1
Log in to reply
Let P(x)=ax5+bx4+cx3+dx2+ex+f. Note that the roots of x2−x+1 are e3iπ and e3−iπ. Thus, P(x) is divisible by x2−x+1 if P(e3iπ)=0 and P(e3−iπ)=0. From this, we gather that a+b=d+e and 2a−2b−c−2d+2e+f=0, which implies e+2f+a=b+2c+d. Also, since a+b=d+e, a−d=e−b=c−f.
We now consider P(x)=(k+l)x5+9x4+(m+l)x3+kx2+(9+l)x+m, with l>0 and k≤9≤m. For a given l, there are (39−k) ways of choosing k, m and n such that l+m≤9. Now since the coefficients are distinct, we must subtract any combination with the difference in k, m and n being l.
There are 9−2l ways to select two numbers differing by l and 7−l ways to select the remaining numbers. This implies that for a given l, there are (39−l)−(9−2l)(7−l)+9−3l polynomials satisfying, which adds up to 53. However, we take into account the rearrangements of the coefficients (there are 12 for each) so there are 53×12=636 polynomials satisfying.
Problem 11
Let f be a function f:Z→Z that satisfies below:
f(y+f(x))=f(x−y)+y(f(y+1)−f(y−1))
Find all functions that satisfy.
Log in to reply
Put y=0. f (f (x))=f (x)
This implies either f (x) is a constant function or f (x)=x.
Log in to reply
Its a constant function
And cannot be f(x)=x
Log in to reply
RHS becomes x-y+y (y+1-y+1)=x+y
Clearly f (x)=x satisfies
Alternatively take inverse of my equation, you will get f (x)=x
Log in to reply
Log in to reply
Not necessarily.
Log in to reply
One more: f (x)=|x|, one more f (x)=-|x|
Here are the solutions:
1) f(x)=a if x is even and a is even, and f(x)=b if x is odd and b is odd.
2) f(x)=c∀c
3)f(x)=x∀x
Sharky discussed some time back this with me so it would be unfair for me to post the solution.
Log in to reply
Well I found these solutions but couldn't find a way to prove that these were the only ones, so I gave up. I'm just waiting for the solution now. (is it some kind of induction?)
Ok the final answer
As I have wrote f (f (x))=f (x)
The following satisfy
Constant function
f is odd when x is odd and f is even when x is even
f (x)=x
Your time is up. 24 hours were given to solve the problem and 2 hours to post the next one for you. 26 hours are over, so by point number 5 since I am creator of problem 10, I can post problem 12!
Log in to reply
Sorry, I was busy, I'll post solution ASAP.
PROBLEM 12
For the equation x2+bx+c, as x approches ∞, it also approaches ∞.
For the equation ∣−x2+bx+c∣, as x approaches ∞, it also approaches ∞
Can we compare the 2 infinities? If yes, which is greater?
Log in to reply
Yes, we can compare the two infinities. If b is +ve then x^2+bx+c is greater and if b is -ve then |-x^2+bx+c|is greater.
Log in to reply
Right post next
Problem 13
In problem 12, if c approaches −∞, such that when x tends to ∞, x2+bx+c tends to 0.
Which of the following can be the value of ∣−x2+bx+c∣?(b=0)
A) 0
B) ∞
C) −∞
D)20162015
Log in to reply
B
Log in to reply
Wrong. Think again
B) and D) both can be correct. Check your question again unless it is a multiple options correct question.
Log in to reply
Aman you are right. Post the next. I created this question as multicorrect only!
And are you guys kidding? Why three contests are going on simultaneously?
Log in to reply
Geometry one is stucked, JEE algebra one I told Rajdeep not to start but he started!
Your time limit to create a question is up. Read point 5.
PROBLEM 14
Which of the following sets are incomparable
A). Set of all living children, set of all living humans
B). Set of regular polygons, Set of irregular polygons
C). Set of all the triangles in a plane, set of all the squares in a plane
D). Set of all the things in this universe,set of all the chairs.
Question
You have to explain why all the others are wrong and this is right (Your answer)
Log in to reply
Firstly, in A), we have the the set of all living children is a subset of the set of all living humans, so they can be compared.
Secondly, in C), we have the set of all triangles, which is an uncountable infinite, versus the set of all squares, which is also an uncountable infinite, and we cannot compare these two sets.
Thirdly, in D), we has the set of all chairs is a subset of the set of all things in the universe, so they can be compared.
Finally, in B), we have the set of regular polygons is disjoint from the set of irregular polygons. However, note that the probability an arbitrary polygon is regular is 0 (not impossible, 0), so it is almost guaranteed that an arbitrary polygon is irregular. Thus, we must have the set of all regular polygons to be less than the set of irregular polygons.
My answer is C).
Problem 15
Find the least real number r such that for each triangle with side lengths a, b, c,
3a3+b3+c3+3abcmax(a,b,c)<r
Log in to reply
WLOG, it can be assumed that a≥b≥c
Using the triangle inequality, and some trivially true statements:
b+c−a>0
b2+c2+a2+ab+ac−bc>0
b3+c3−a3+3abc>0
a3+b3+c3+3abc>2a3
The rest should follow after rearranging...
a3+b3+c3+3abca3<21
3a3+b3+c3+3abca<321
Thus, r=321.
Problem 16
Evaluate: n=0∑∞cosnθcosnθ
Assume θ is such that the sum converges.
Log in to reply
is it ? i replaced theta with x 21.cos2xeix−cosx−cosxe2ix+eix2eix−cosx−cosxe2ix
Log in to reply
It is assumed that I want the answer in the simplest possible form...
I will prove the fraction simplifies to 2. We are required to prove:
cos2xeix−cosx−cosxe2ix+eix2eix−cosx−cosxe2ix2eix−cosx−cosxe2ixcosxe2ix+cosxcosx(2cos2x+2isinxcosx)2cos2x(cosx+isinx)=2=2cos2xeix−2cosx−2cosxe2ix+2eix=2cos2xeix=2cos2x(cosx+isinx)=2cos2x(cosx+isinx)
which is clearly true. Thus, the original fraction must be equal to 2, so the value of the summation is 1.
Problem 18
Since @Archit Agrawal's time ran out, I'll post next question.
Consider the sequence
an=2−n2+n4+411,n≥1
Prove that a1+a2+…+a119 is an integer and find the value of it.
Log in to reply
168
Log in to reply
Can you prove it? That was the question asked.
Here is the solution:
We will take the conjugate of the expression to get as follows:
an=2−n2+n4+411=2−n4−(n4+41)n2−n4+41=2+4n2−24n4+1
We can factorise 4n4+1 as (2n2−2n+1)(2n2+2n+1). Note that (2n2−2n+1)+(2n2+2n+1)=4n2+2. Thus, this expression is the perfect square identity. We have
anan=4n2+2−24n4+1=(2n2+2n+1)2+(2n2−2n+1)2−2×2n2+2n+1×2n2−2n+1=(2n2+2n+1−2n2−2n+1)2=2n2+2n+1−2n2−2n+1
Note that this is a telescoping series so almost all terms cancel out in the, so we are left with 2×1192+2×119+1−2×12−2×1+1=169−1=168, which is an integer. Thus proven.
Problem 19
Let a,b,c>0 be real parameters. Solve the system of equations:
ax−by+xy1bz−cx+zx1cy−az+yz1=c=a=b
Log in to reply
This solution is a bit messy, but is quite quick as opposed to the other possible solutions (I think).
Consider a, b and c as the unknowns instead, so we get a set of linear equations. If we multiply the first equation by x, and then y and substitute cx in the second question and cy in the last one, we obtain
a(x2+1)−b(xy+z)a(xy−z)−b(y2+1)=xz1−y1(i)=−x1−yz1(ii)
Multiply equation (i) by y2+1 and equation (ii) by −(xy+z), and add them up. It follows that
a(x2+y2+z2+1)=xzx2+y2+z2+1
so a=xz1. Similarly, b=yz1 and c=zx1. Thus, abc=(xyz)21, so xyz=±abc1. From this, we can attain the solutions of the system are
(abcb,abca,abcc),(abc−b,abc−a,abc−c)
Log in to reply
I made a silly mistake while calculating by matrices... Now I got it haha
Problem 20
Let x1,x2,…,xn be the roots of the polynomial Xn+Xn−1+…+X+1. Prove that
k=1∑n1−xk1=2n
Log in to reply
k=1∑n1−xk1=(1−x1)(1−x2)⋯(1−xn)[(1−x1)(1−x2)⋯(1−xn−1)]+⋯+[(1−x2)(1−x3)⋯(1−xn)] (Numerator has total of n addends, each missing different 1−xk)
After multiplying out the products we get 1−(x1+⋯+xn)+...+(−1)n×(x1x2⋯xn)n×1−(n−1)×(x1+⋯+xn)+⋯−(−1)n×1×(x1x2⋯xn−1+⋯+x2⋯xn)
After using Vieta's formulas we get n+12n(n+1)=2n.
Q.E.D
Log in to reply
Nice solution.
Post the next problem.
Good solution. You can post the next problem, but see if you can get a simpler, non-expansive solution.
Problem 21
Let a,b,c be positive real numbers such that 27ab+2bc+18ca=81. Prove that
a1+b2+c3≥3
Log in to reply
According to Mathematica, the inequality is stricter than the one you have provided us with. Mathematica states that the expression is greater than or equal to 3377, which is approximately 3.56
Log in to reply
You can post next
Please post next
Log in to reply
I give my right to post to everyone.
Here's the solution:
a1+b2+c3=2a1+2b1+2b3+4c9+4c3+2a1≥22a1⋅2b1+22b3⋅4c9+24c3⋅2a1=ab1+2bc27+2ca3≥3⋅3ab1⋅2bc27⋅2ca3=3⋅3ab1⋅32bc27⋅32ca3≥3⋅ab+272bc+32ca3=3⋅27ab+2bc+18ca81=3. □
Log in to reply
I did not see that approach at all! Great job!
Problem 23
Since Jesse Nieminen didn't post, my turn again!
Let a,b,c,d∈R+ and a+b+c+d+abcd=5. Prove
a1+b1+c1+d1≥4
Log in to reply
Post next. Time elapsed
Log in to reply
This competition is now me solving or not solving problems by @Sharky Kesa :D
I have a solution using Lagrange Multipliers :P
Log in to reply
Log in to reply
Problem 24
Prove 21/x<xx+1 for x>1.
Log in to reply
Counter-example: x=50.
Log in to reply
LOL, sorry, the other way!
Why did no one give the quick and simple solution?!
21/x2xx2xx0<xx+1<(x+1)x<xx+x⋅xx−1+{ Stuff }<{Stuff}
Since the value of Stuff≥1, the final inequality must be true, so the original inequality is true. Thus, proven.
Log in to reply
I was waiting for someone else to post a solution. I solved this in first 5 minutes.
Time elapsed post next
Problem 25
Prove (a1+b1)(a2+b2)…(an+bn)≤(a1+bσ(1))(a2+bσ(2))…(an+bσ(n)) where {a} and {b} are similarly ordered sequences and σ denotes a permutation.
Log in to reply
Permutations are always greater than or equal to 1 so each bracket of lhs is smaller than corresponding bracket of rhs
Log in to reply
No, what is meant is that the permutation is a permutation of the set 1,2,3,4,...,n. e.g. if n=3, we can assume a permutation of it to be 2,1,3, so σ(1)=2, σ(2)=1 and σ(3)=3. Note that the above must hold true for any permutation.
Furthermore, even if permutation is defined as what you think it was, you're wrong. What if b was negative?
Time is up bro..